Mathematics
Mathematics, 05.06.2020 18:57, golffuture666

Prove that:
(tan20°)^2+(tan40°^)2+(tan80°)^2=33

answer
Answers: 1

Other questions on the subject: Mathematics

image
Mathematics, 21.06.2019 19:50, electrofy456
What is the result of the following division?
Answers: 1
image
Mathematics, 21.06.2019 20:30, plzhelpmeasap46
Acompany produces packets of soap powder labeled “giant size 32 ounces.” the actual weight of soap powder in such a box has a normal distribution with a mean of 33 oz and a standard deviation of 0.7 oz. to avoid having dissatisfied customers, the company says a box of soap is considered underweight if it weighs less than 32 oz. to avoid losing money, it labels the top 5% (the heaviest 5%) overweight. how heavy does a box have to be for it to be labeled overweight?
Answers: 3
image
Mathematics, 21.06.2019 21:00, natjojo0512
Rewrite the following quadratic functions in intercept or factored form. show your work. f(x) = 3x^2 - 12
Answers: 1
image
Mathematics, 21.06.2019 21:30, juniieb
Atrain leaves little? rock, arkansas, and travels north at 90 kilometers per hour. another train leaves at the same time and travels south at 80 kilometers per hour. how long will it take before they are 680 kilometers? apart?
Answers: 1
Do you know the correct answer?
Prove that:
(tan20°)^2+(tan40°^)2+(tan80°)^2=33...

Questions in other subjects:

Konu
History, 10.09.2019 23:20
Konu
History, 10.09.2019 23:20